LSAT and Law School Admissions Forum

Get expert LSAT preparation and law school admissions advice from PowerScore Test Preparation.

 Administrator
PowerScore Staff
  • PowerScore Staff
  • Posts: 8917
  • Joined: Feb 02, 2011
|
#22734
Complete Question Explanation

Main Point. The correct answer choice is (E)

If personal income tax cuts are made across the board, then one of two things happens:
Either non-progressive taxes have to be raised (favoring the wealthy), or a deficit occurs, driving up interest rates (again favoring the wealthy). The conclusion to this argument is presented at the end of the opening sentence: “across-the-board cuts (in personal income tax rates) tend to have a similar (wealth-biased) effect.”

Answer choice (A): The author makes this point at the opening of the passage, but it is not the main conclusion of the argument, so this answer choice is incorrect.

Answer choice (B): While this may be accurate, it certainly does not reflect the main point of the stimulus, which is that across-the-board cuts disproportionately benefit the wealthy.

Answer choice (C): The wealthy enjoy greater benefit in various contexts, but the main point of this passage is that across-the-board income tax cuts favor the wealthy.

Answer choice (D): This is another answer choice which is factually accurate, but since it doesn’t reflect the main point of the passage, it is incorrect.

Answer choice (E): This is the correct answer choice.
 Sherry001
  • Posts: 81
  • Joined: Aug 18, 2014
|
#20073
Hello ;
I had trouble choosing between A and E. Super lost ! Super confusing stimulus . I read it over and over again just to get an "okay " understanding :(

Is A wrong because it says , "cuts benefit the wealthy more than they benefit others"? Kind of implying that they both get benefits and not really covering the " Disproportionate" part ?

And is E correct because " generally the wealthy benefit more " then we can infer that the non wealthy are less likely to benefit from these cuts ? So hinging on disproportionate ?
 Robert Carroll
PowerScore Staff
  • PowerScore Staff
  • Posts: 1787
  • Joined: Dec 06, 2013
|
#20080
Sherry,

The problem is actually that answer choice (A) talks about "cuts...for upper income brackets" whereas answer choice (E) talks about "across-the-board...cuts." The former point was granted at the beginning of the first sentence, but the end of that sentence contains the main point that the economist attempts to demonstrate with the rest of the information given.

Robert Carroll
 andriana.caban
  • Posts: 142
  • Joined: Jun 23, 2017
|
#36356
Hi!

I got E, but I was still unsure of the reasoning. I got that answer by doing process of elimination, while the other choices were still correct, they were also similar to each other. Answer choice E reflected the authors main argument, and was different from the other answer choices. I have a question with my prephrasing.

I'm currently using sticky notes to cover answer choices and write my prephrase on a sticky note and what I wrote was: " Tax cuts favor the wealthy but if there are no tax cuts, interest rates increase because the government budget increase, which ultimately favors the weathly"

Am I supposed to be generalize the stimulus or trying to answer the question in the prephrasing step?
 nicholaspavic
PowerScore Staff
  • PowerScore Staff
  • Posts: 271
  • Joined: Jun 12, 2017
|
#36401
andriana.caban wrote:Hi!

I got E, but I was still unsure of the reasoning. I got that answer by doing process of elimination, while the other choices were still correct, they were also similar to each other. Answer choice E reflected the authors main argument, and was different from the other answer choices. I have a question with my prephrasing.

I'm currently using sticky notes to cover answer choices and write my prephrase on a sticky note and what I wrote was: " Tax cuts favor the wealthy but if there are no tax cuts, interest rates increase because the government budget increase, which ultimately favors the weathly"

Am I supposed to be generalize the stimulus or trying to answer the question in the prephrasing step?
Hi adriana,

Excellent question. :-D As an initial matter, remember with Main Point questions, not only must the answer you select follow from the information in the stimulus, but it must also accurately summarize the author's point of view. If an answer choice is true according to the stimulus but fails to address the main point, it is incorrect.

With that said, it helps to focus on the conclusion in the stimulus in order to understand the Main Point and for the sake of a good prephrase before looking at the answer choices.

Remember that with these types of questions, the conclusion is often at the beginning or in the middle, but not usually at the very end. In this stimulus, the conclusion is actually at the end of the first sentence - "across-the-board cuts for all brackets tend to have a similar effect (of disproportionately benefitting the wealthy)." From that conclusion, the Economist then offers a series of arguments intended to support it. So your prephrase should follow that Economist's conclusion very closely but only after you have read the stimulus and question stem first. So while several incorrect answer choices may accurately restate a portion of the stimuli, ony (E) actually correctly summarizes the Economist's conclusion thereby making it the correct answer.

Thanks for the great question!
 Alexis
  • Posts: 7
  • Joined: Oct 26, 2017
|
#40951
Hi,

I had a bit of trouble determining what the conclusion was for this question. Could you perhaps walk me through the process of how you were able to differentiate between the conclusion and the rest of the stimulus?

Thanks!
Alexis
 James Finch
PowerScore Staff
  • PowerScore Staff
  • Posts: 943
  • Joined: Sep 06, 2017
|
#40968
Hi Alexis,

The stimulus in this question can be difficult to parse because, while the structure is straightforward, the very specific topic and language involved requires careful reading. As the last question in the section, the examiners are relying on the time crunch many test takers will be feeling at this point to make this question more difficult.

Starting from the first sentence, the stimulus gives us two scenarios, claiming that both benefit the rich: cuts in income tax rates in upper income brackets, and cuts across all income brackets. There are a couple linguistic clues in this sentence as to the role it will play in the stimulus's argument. First, we are told that it is "obvious" that cuts in upper income brackets disproportionately benefit the wealthy, which is a clue that this information will likely be extraneous. Then we are told that across-the-board cuts also benefit the wealthy disproportionately, information that is presumably not obvious, and thus requires an argument to prove. So that is a hint that this claim is some form of conclusion, either main or intermediate.

Reading on, the next sentence explains how across-the-board income tax cuts would lead to greater reliance on nonprogressive taxes, which would benefit the wealthy, providing evidence for the claim made in the first sentence. That claim is then further supported by the next (third) sentence, which tells us that even if nonprogressive taxes aren't raised, the across-the-board income tax cuts increase the budget deficit and driving up interest rates. Lastly, the stimulus tells us that driving up interest rates benefits the wealthy, as they are the people with money to lend. Put together, we can see that the last three sentences serve as supporting evidence for the claim about across-the-board income tax cuts made in the first sentence. This makes that claim our main conclusion, and what (E) restates.

I hope this clears things up!

Get the most out of your LSAT Prep Plus subscription.

Analyze and track your performance with our Testing and Analytics Package.